Attending Class The following data represent the number of days absent, x, and the final grade, y, for a sample of college students in a general education course at a large state University
No. of Absensense, x 0 1 2 3 4 5 6 7 8 9
Final grade, y 89.2 86.4 83.5 81.1 78.2 73.9 64.3 71.8 65.5 66.2
A) Find the least-sqaures regresssion line treating number of absenses as the explanatory variable and final grade as a response variable.
B) Interpret the slope and y-intercept , if appropriate.
C) Predict the final grade for a student who misses five class periods and compute the residual. Is the final grade abover or below average for this number of absenses.
D) Draw a least-sqaures regression line on the scatter diagram of the data.
E) Would it be reasonable to to use the least-sqaures regression line to predict the final grade for a student who has missed 15 class periods? why or why not?

Answers

Answer 1

We get the following answers:

A) The least square regression line is:

yi = 88.73 - 2.827(xi)

B) The slope  is β₁ = -2.827 and The intercept is β₀ = 88.73.

C) e*₆ = -0.6964, final grade is below average grade.

E) It would not be reasonable since no of absence increases the score decreases, it works well for small no of absences but will not work for large number of absences since it may give absurd final grade.

we have Xi = No of absences

Yi = Final grade

The regression model is

Yi = β₀ +  β₁Xi + i

β₀ = y -  β₁x

we have n = 10, x bar = 4.5 , y bar = 76.01

cov(x,y) = -23.325 and var(x) = 9.1667

therefore,  β₀ = 88.73 and  β₁ = -2.827

A) Least square regression line is given by:

yi =  β₀+ β₁Xi

yi = 88.73 - 2.827(xi)

B) The slope  β₁ = -2.827 which means when no of absence is increased by 1 duty then the final grade will decrease by 2.827

The intercept  β₀ = 88.73 when there are 0 days absence the average grade is 88.73

C) Predicted graph for student who misses five class periods is

x = 5

y*5 = 88.73 - 2.827(5) = 74.5964

Hence for corresponding residual is:

e*₆ = 73.9 - 74.5964

e*₆ = -0.6964, final grade is below average grade.

D) please see graph

E) when student miss 15 class periods the final grade will be

X = 5

y* = 88.73 - 2.827(15)

= 46

It would not be reasonable since no of absence increases the score decreases, it works well for small no of absences but will not work for large number of absences since it may give absurd final grade.

Learn more about Least square regression here:

brainly.com/question/14563186

#SPJ4


Related Questions

Find the next permutation in lexicographic order About For each permutation of (1,2,3,4,5,6,7,give the next largest permutation or indicate that the permutation is the last one in lexicographic order (1,2,3,4,5,6,7) (7.6,5,4,3,2, 1) (1.7,6,4,2,3,5) (3,7,6,5,4,2,1) (5,4,7,6,3,2, 1)

Answers

On solving the provided question, we can say that In lexicographic order, the following variation will be (5,6,7,4,3,2,1).

What is a permutation?

It is known as permuting the set in mathematics to rearrange the components of a set that is already ordered or, if the set is not already ordered, to place its members in a linear or sequential order. In this sense, "permutation" refers to the process of changing the linear order of an ordered set.It should be mentioned that a permutation is a mathematical concept that describes several potential arrangements of numbers or other objects.

The ascending numerical order is the lexicographic order when it comes to numbers. From left to right, the numbers are growing. The permutations of "1,2,3," for instance, are 123, 132, 213, 231, 312, and 321 in lexicographic order.

In lexicographic order, the following variation will be (5,6,7,4,3,2,1).

To know more about permutation visit:
https://brainly.com/question/1216161

#SPJ4

If t less than s and s less than r what is the relationship between the values t and r?

Answers

If t is less than s and s is less than r then the relationship between the values t and r is - t is less than r.

We are given that-

t is less than s i.e. t < s

and

s is less than r i.e. s < r

Now, let us find the relationship between the values of t and r.

t < s and s < r

this implies that -  t < s < r

Hence, we get t < r.

Thus, we get that if t is less than s and s is less than r then the relationship between the values t and r is - t is less than r i.e. t < r.

Read more about inequalities:

brainly.com/question/28230245

#SPJ4

there are $320 available to fence in a rectangular garden. the fencing for the side of the garden facing the road cost $6 per foot and the fencing for the other three sides is $2 per foot (see picture). a. determine the objective and constraint equations b. express the quantity to be maximized as a function of x. c. find the optimal values of x and y.

Answers

To answer this question, we first need to determine the objective and constraint equations.

The objective is to maximize the area of the rectangular garden, which is given by the equation $A = xy$.

The constraint is that the total cost of the fencing must not exceed $320. In other words, we have the equation $6x + 2(y+x) \le 320$.

To express the quantity to be maximized as a function of $x$, we can simply plug in the equation for the area of the rectangular garden into the objective equation. This gives us the function $f(x) = xy = x^2y$.

To find the optimal values of $x$ and $y$, we can use the method of Lagrange multipliers. Let $\lambda$ be the Lagrange multiplier. The Lagrange function is given by

$$L(x,y,\lambda) = x^2y + \lambda(6x + 2(y+x) - 320)$$

To find the optimal values of $x$ and $y$, we need to find the values of $x$ and $y$ that maximize $L(x,y,\lambda)$. To do this, we take the partial derivatives of $L(x,y,\lambda)$ with respect to $x$ and $y$ and set them equal to 0. This gives us the equations

$$\frac{\partial L}{\partial x} = 2xy + 6\lambda = 0 \quad \text{and} \quad \frac{\partial L}{\partial y} = x^2 + 2\lambda = 0$$

We can solve these equations simultaneously to find the optimal values of $x$ and $y$. First, we can solve for $y$ in the second equation to get

$$y = -\frac{x^2}{2\lambda}$$

Substituting this expression for $y$ into the first equation, we get

$$2x \left(-\frac{x^2}{2\lambda}\right) + 6\lambda = 0 \quad \Rightarrow \quad -x^3 + 12\lambda^2 = 0$$

Since $\lambda$ cannot be 0, we can divide both sides of this equation by $-12\lambda^2$ to get

$$x^3 = 12\lambda^2 \quad \Rightarrow \quad x = \sqrt[3]{12\lambda^2}$$

Substituting this expression for $x$ into the expression for $y$, we get

$$y = -\frac{\sqrt[3]{12\lambda^2}}{2\lambda}$$

Now we need to find the value of $\lambda$ that satisfies the constraint $6x + 2(y+x) \le 320$. Substituting the expressions for $x$ and $y$ into this equation, we get

$$6\sqrt[3]{12\lambda^2} + 2\left(-\frac{\sqrt[3]{12\lambda^2}}{2\lambda} + \sqrt[3]{12\lambda^2}\right) \le 320$$

Simplifying, we get

$$-\frac{\sqrt[3]{12\lambda^2}}{2\lambda} + 10\sqrt[3]{12\lambda^2} \le 320$$

Dividing both sides by $10\sqrt[3]{12\lambda^

To find the circumference of a circle, the measurement of the circle’s:

radius or diameter must be known.

area must be known.

base or height must be known.

tangent must be known.

None of these choices are correct.

Answers

Answer: Radius or Diameter

Step-by-step explanation: The formula for circumference is diameter times pi. Since radius is half of diameter, you could still easily find the circumference. You could also find it with area, but that involves multiple formulas and a lot of math. I hope this helped :)

Four-Sided Dice A pair of four-sided dice-each with the numbers from 1 to 4 on their sides- -are rolled, and the num bers facing down are observed. (Note: A four-sided die is sim- ilar to a pyramid with a triangular base.) (a) List the sample space. (b) Describe each of the following events as a subset of the sample space i) Both numbers are even. (ii) At least one number is odd. iii Neither number is less than or equal to 2. (iv) The sum of the numbers is 7. (v) The sum of the numbers is greater than or equal to 6. (vi) The numbers are the same. (vii) A 2 or 3 occurs, but not both 2 and 3. (viii) No 4 appears.
Previous question

Answers

Answer:

Step-by-step explanation:

(a)

The sample space is

{(1,1), (2,2), (3,3), (4,4), (1,2), (1.3), (1,4), (2,1), (2,3), (2,4), (3,1), (3,2) (3,4), (4,1),(4,2), (4.3)}

(b)

i  {(2,2), (4,4), (2,4), 4,2)}

ii {(1,1), (1,2), (3,3), (1,3), (1,4), (2,1), (2,3), (3,1), (3,2), (3,4), (4,1), (4,3)}}

iii {(1,1)}

iv {3,4), (4,3)}

v {(3,3), (2,4), (4,2), (3,4), (4, 3), (4,4)}

vi {(1,1), (2,2), (3,3), (4,4)}

vii  {(1,2), (1,3), (2,1), (3,1), (2,2), (2,4), (3,3), (3,4), (4,2), (4,3)}

viii {(1,1), (2,2), (3,3), (1,2), (1.3), (2,1), (2,3), (3,1), (3,2)}

Suppose that the probability that the dart hit the area between the square of side length 14 and the square of side length 16

Answers

The probability of the area hit by the dart between the bigger square and the smaller square is equal to 0.7656.

As given in the question,

Total number of squares present = 2

Side length of the bigger square = 16 units

Side length of the smaller square = 14 units

Area of the bigger square = ( side length )²

                                           = ( 16 )²

                                           = 256 square units

Area of the smaller square = ( side length )²

                                           = ( 14 )²

                                           = 196 square units

Here favorable outcome is represented by small square and total outcome is represented by bigger square

Probability that the given dart hit the area between the square is

= 196 / 256

= 49/64

= 0.765625

≈ 0.7656

Therefore, the probability of the dart hitting the area formed by bigger square and smaller square is equal to 0.7656.

Learn more about probability here

brainly.com/question/30034780

#SPJ4

write the equation of the parabola in vertex form that has the following information: vertex: (2, -8) directrix: x

Answers

The Equation of parabola  ( y + 8 )² = 12( x - 2 )

In math, what is a parabola?

A parabola is a U-shaped plane curve in which every point is situated at an equal distance from both the focus, a fixed point, and the directrix, a fixed line.

                 All of the parabola-related ideas are discussed here because it is a crucial component of the conic section topic.

Given,

Directrix is known as x=a Directrix=a=3 Equation of parabola

(Y-y1)²= 4a(X-x1)

Vertex (X1,Y1) = (2,-8)

Directrix=a=3

Equation of parabola

       ( y - y₁ )² = 4a( x - x₁)

        ( y - ( -8))² = 4 * 3(x - 2 )

          ( y + 8 )² = 12( x - 2 )

Learn more about parabola

brainly.com/question/21685473

#SPJ4

please help me please #1

Answers

Answer:

  14.7

Step-by-step explanation:

You want the side opposite the 60° angle in a right triangle with hypotenuse 17.

Sine

The sine relation is ...

  Sin = Opposite/Hypotenuse

Solving for the opposite side gives ...

  Opposite = Hypotenuse × Sin

  x = 17·sin(60°) ≈ 14.7

The missing side is about 14.7.

Use Taylor's Inequality to determine the number of terms of the Maclaurin series for e^x that should be used to estimate e^0.1 to within 0.0000001.

Answers

The number of terms of the Maclaurin series for eˣ that should be used to estimate [tex]e^{0.1}[/tex] to within 0.00001 is 4.

Taylor’s inequality:

If Rn( x ) = 0

Rn(x)=0

Rn​(x)=0.

The given function is eˣ

Thus, all the derivatives of eˣ is eˣ itself.

Suppose that, the nth derivative of eˣ is eˣ

To estimate the value of [tex]e^{0.1}[/tex] substitute the value of x as 0.1.

Note that Taylor series is called Maclaurin series when a=0.

Usually, Maclaurin series will starts with n=2.

Increase the value of n until the reminder is less than 0.00001.

By Taylor’s inequality,

If | [tex]f^{n}[/tex](0.1) | = [tex]e^{0.1}[/tex]≤ M then | Rₙ(x) |≤[tex]\frac{M}{(n+1)!}[/tex] [tex]|0.1-0|^{n+1}[/tex]

Substitute the value of n as 2. Then,

[tex]|R2(x)|[/tex]≤[tex]\frac{M}{(2+1)!} |0.1-0|^{2+1}[/tex]

≤ [tex]\frac{e^{0.1}}{3!} |0.1|^{3}[/tex]

≤ [tex]\frac{e^{0.1}}{6} 0.001[/tex]

≤ 0.00018

Since [tex]|R2(x)|[/tex] ≥ choose the value of n as 3. Then,

[tex]|R3(x)|[/tex] ≤[tex]\frac{M}{(3+1)!} |0.1-0|^{3+1}[/tex]

≤ [tex]\frac{e^{0.1}}{4!} |0.1|^{4}[/tex]

≤ [tex]\frac{e^{0.1}}{24} 0.0001[/tex]

≤ 0.0000046

Therefore, [tex]|R3(x)|[/tex] ≤ 0.00001

That is, n=3 satisfies the inequality.

So, by adding the 4 terms (when n=0,1,2,and 3) of the Maclaurin series, it

is possible to estimate the value of [tex]e^{0.1}[/tex] to within 0.00001.

Thus, the number of terms of the Maclaurin series for eˣ that should be used to estimate [tex]e^{0.1}[/tex] to within 0.00001 is 4.

The number of terms of the Maclaurin series for eˣ that should be used to estimate [tex]e^{0.1}[/tex] to within 0.00001 is 4.

To learn more about  Maclaurin series visit:

brainly.com/question/24188694

#SPJ4

What is the range of the data set?
{6.2,9.1,2.9,1.6,9.3,5.5,8.3}
Enter your answer as a decimal, rounded to the nearest tenth, like this: 4.2

Answers

The range of the data set { 6.2 , 9.1 , 2.9 , 1.6 ,9.3 , 5.5 , 8.3 } is 7.7 .

Given :

the range of the data set {6.2,9.1,2.9,1.6,9.3,5.5,8.3}

Enter your answer as a decimal, rounded to the nearest tenth .

Range :

In statistics, the range is the spread of your data from the lowest to the highest value in the distribution.

Range = highest value - lowest value

Arrange in ascending order = { 1.6 , 2.9 , 5.5 , 6.2 , 8.3 , 9.1 , 9.3 }

Here highest value = 9.3

Lowest value = 1.6

Range = 9.3 - 1.6

= 7.7

Learn more about the range here:

https://brainly.com/question/28135761

#SPJ4

what is the answer to the question below

Answers

The equation of the area is 308 = 2 * (10 * x) + 2 * (2 * x) + 2 * (10 * 2) + 6 * (5 * 5) - 2 * (5 * 5)

How to determine the equation?

From the question, we have the following parameters that can be used in our computation:

Area = 308

The formula used to calculate area

Area = S.A of the bottom + S.A of the top - 2 * Overlap

Using the given figure, we have

S.A of the top = 6 * (5 * 5)

S.A of the bottom = 2 * (10 * x) + 2 * (2 * x) + 2 * (10 * 2)

Overlap = 2 * (5 * 5)

So, we have

308 = 2 * (10 * x) + 2 * (2 * x) + 2 * (10 * 2) + 6 * (5 * 5) - 2 * (5 * 5)

Hence, the equation is (b)

Read more about surface area at

https://brainly.com/question/15990299

#SPJ1

Determine the component representation and magnitude of the vector having the initial point P and the terminal point Q.
P(-3,2) and Q(5,17)
The component representation of the vector having the initial point P and the terminal point Q is....
The magnitude of the vector having the initial point P and the terminal point Q is ....

Answers

The magnitude of the vector having the initial point P and the terminal point Q is 17.

What is magnitude ?

Simply said, "distance or amount" is the definition of size. In terms of motion, it shows the absolute or relative size, direction, or movement of an item. It is used to describe something's size or scope. Magnitude in physics often refers to a size or amount.

Given: p (- 3,2) , q (5,17)

vector  = (17-2)i + (5+3)j

= 15i + 8j

Magnitude = [tex]\sqrt{(15^{2}+8^{2} ) }[/tex]

magnitude =17

To learn more about magnitude checkout https://brainly.com/question/15681399

#SPJ4

elodie is putting on a fashion show and has five fabulous outfits for her five fabulous fashion models. however, on the day of the show, two of the outfits were ruined in an unfortunate permanent marker incident. regardless, the show must go on and the remaining outfits will be presented. if each outfit can only be worn by one model and there is no time for any model to wear more than one dress, how many different shows can elodie put on? (note: two shows are considered the same if they contain the same models wearing the same dresses.)

Answers

By analyzing the question, we find that there are a total of 5 * 4 * 3 = 60 different shows that Elodie can put on.

What is permutation?

A permutation is an arrangement of a set of items in a specific order. Permutations are typically denoted using the notation "P" followed by the number of items being permuted and the number of items being chosen at a time.

What is the formula to calculate permutation?

The formula for calculating permutations is as follows:

P(n,r) = n! / (n-r)!

where:

P(n,r) is the number of permutations of r items from a set of n items.

n! is the factorial of n, which is the product of all positive integers less than or equal to n.

There are five fashion models and three remaining outfits, which means there are 5 ways to choose the model who wears the first outfit, 4 ways to choose the model who wears the second outfit, and 3 ways to choose the model who wears the third outfit. Therefore, there are a total of 5 * 4 * 3 = 60 different shows that Elodie can put on.

To learn more about permutation visit:

https://brainly.com/question/1216161

#SPJ4

ABCD is a rectangle find the length of AC and find the measures of a and 0

Answers

In the given triangle, length of AC = 12.3 units

α= 45⁰ and θ = 45⁰

Using Pythagoras theorem,

[tex]AC^{2} = AB^{2} + BC^{2}[/tex]

[tex]AC^{2}[/tex] = 9 + 144

AC = 12.37

Now, to find the angles:

AC is the diagonal.

Then AD=BC and AB=CD

And ∠ABC=∠BCA=∠CDA=∠DAB=90⁰

In ΔABC and ΔADC

∠ABC=∠ADC (Angle of rectangle)

AB=DC (Opposite side of rectangle)

AC=AC (Common side)

∴ΔABC≅ΔADC

∴∠ACD=∠ACB

∵∠ACD+∠ACB=90⁰ ..........[Angle ACB is the angle of rectangle]

∴2∠ACD=90⁰

⇒∠ACD=45⁰

⇒θ = 45⁰

similarly, α = 45⁰

Pythagoras theorem:

A right triangle's three sides are related in Euclidean geometry by the Pythagorean theorem, also known as Pythagoras' theorem. According to this rule, the areas of the squares on the other two sides add up to the area of the square whose side is the hypotenuse, or the side across from the right angle.

This theorem can be written as an equation relating the lengths of the sides a, b and the hypotenuse c, often called the Pythagorean equation:

[tex]a^{2} + b^{2} = c^{2}[/tex]

To know more about Pythagoras theorem:

https://brainly.com/question/343682

#SPJ1

Solve and graph the inequality on a number line:
42>g+27

Answers

The solution of the inequality 42 > g + 27 is 25 > g  and the graph of the inequality has been plotted on a number line

The given inequality is

42 > g + 27

The inequality is the mathematical statement that shows the relationship between two expression with and inequality sign. The inequality maybe less than, less than or equal, greater than, greater than or equal etc. The equal sign will not be a part of inequality

Th given inequality is

42 > g + 27

Subtract both side of the inequality by 27

42 - 27 > g + 27 - 27

25 > g

Plot the inequality in number line

Therefore, the solution is 25 > g and the inequality has been plotted on the number line

Learn more about inequality here

brainly.com/question/20383699

#SPJ4

a census is a statistical study in which an entire population is counted, not just a sample. which of the following stats is most likely to be based on a census?

Answers

true census  is most likely to be based on a census There are many benefits to choosing a portion of the population selected to represent the population.

Reducing the cost and time needed to conduct research. Researchers will save more time and money analyzing sample than the whole population.  

• Reducing resource deployment. The manpower needed to analyze a sample is less than the manpower needed to analyze the whole population.  

Increasing data accuracy. Data taken from the sample will be more accurate than data taken from the whole population because the sample is more willing to join the research.  

• Creating an intensive and exhaustive data. The data taken from a sample will be more intensive and exhaustive because the respondents are minimum.  

learn more about of census here

https://brainly.com/question/1211838

#SPJ4

round 19.83 to the nearest tenth

Answers

Answer:

19.8

Step-by-step explanation:

19.83 rounded to the nearest tenth is 19.8.

19.8 bc 19.83 gets rounded down to 19.8 bc 3 isn’t greater then 5

Bill has to run some errands around town. Look at the map below to calculate his distance if he starts at home, goes to the video store, then the grocery store, grabs a fast snack, and ends at home. How many units did he walk?

Coordinate grid with points labeled home, video store, fast snacks, and grocery store. Home is located at negative 2, 3, video store is at 4, 3, fast snacks is at negative 2, negative 4, and grocery store is at 4, negative 4.
(4 points)

13 units
22 units
25 units
26 units

Answers

Answer:

26 units

Step-by-step explanation:

First he goes from home to the video store.

home (-2,3) --> video store (4,3)

There is no change in his y distance, so the distance he travels is only the change in x.

distance from home to video store = 4 - (-2) = 6 units

Then he goes from the video store to the grocery store

video store (4,3) --> grocery store (4,-4)

There is no change in his x distance, so the distance he travels is only the change in y.

distance from video store to the grocery store = 3 - (-4) = 7 units

Then he goes from the grocery store to the fast snacks

grocery store (4,-4) --> fast snacks (-2,-4)

There is no change in his y distance, so the distance he travels is only the change in x.

distance from the grocery store to the fast snacks = 4 - (-2) = 6 units

Finally he goes from the fast snack store to his house

fast snacks (2,-4) --> house (2,3)

There is no change in his x distance, so the distance he travels is only the change in y.

distance from the fast snacks to his house = 3 - (-4) = 7 units

therefore the total distance he traveled is 6 units + 7 units + 7 units + 6 units = 26 units

I hope this helps!

21. Which statement is a good definition?
Right angles are angles formed by two intersecting lines.
Skew lines are lines that do not intersect.
A
square is a rectangle with four congruent sides.
Parallel lines are lines that do not intersect.

Answers

The statement that is a good definition is D. Parallel lines are lines that do not intersect.

How to illustrate the information?

A type of quadrilateral with all sides being equal and all angles being 90 degrees is a square.

Lines that do not cross each other but have a constant perpendicular distance are known as parallel lines. Right angles and skew lines are perpendicular to one another but not parallel. Therefore, a square is a type of rectangle in which all four sides are parallel to one another.

Therefore, based on the information, the correct option is D

Learn more about parallel lines on:

https://brainly.com/question/12388786

#SPJ1

Use the Sport Shop data set below to calculate a 3 -year Weighted Moving Average (WMA) to predict the number of sports equipment that will be sold in 2022. Use the weights of 0.5, 0.3, and 0.2 0.5 , 0.3 , and 0.2 for the 3 3 -year WMA, where the first weight is used for the most recent year and the last weight is used for the least recent year. Round your answer to two decimal places, if necessary.

Answers

The 3-year Weighted Moving Average (WMA) for predicting the amount of sports equipment sales in 2022 is 1.7.

What is Weighted Moving Average?

A Weighted Moving Average gives more weight to recent data and less weight to historical data. This is accomplished by multiplying the price of each bar by a weighting factor. WMA will track prices more precisely than a related Simple Moving Average due to its unique formula. A Weighted Moving Average (WMA) is a sort of moving average that prioritizes recent data over historical data. A moving average is a technical indicator that displays how the price has moved on average over a given time period.

Here,

weights=0.5, 0.3, and 0.2

(0.5×1+0.3×2+0.2×3) / (0.5+0.3+0.2) = 1.7

The 3-year Weighted Moving Average (WMA) to predict the number of sports equipment that will be sold in 2022 is 1.7.

To know more about Weighted Moving Average,

https://brainly.com/question/28021669

#SPJ4

BigPhone Company charges a connection fee of $200 and $0.15 per minute. LittlePhone Company charges a connection fee of $175 and $0.18 per minute.

Which phone plan would be cheapest if you spend 310 minutes on the phone?

Group of answer choices

LittlePhone Company is cheaper

Both companies will charge the same amount

BigPhone Company is cheaper

There is not enough information

Answers

The amount of Little phone company is $230.8 and of Big phone company is $246.5. Little Phone Company is cheaper. The correct option is B.

What is an expression?

Expression in maths is defined as the collection of numbers variables and functions by using signs like addition, subtraction, multiplication, and division.

Numbers (constants), variables, operations, functions, brackets, punctuation, and grouping can all be represented by mathematical symbols, which can also be used to indicate the logical syntax's order of operations and other features.

Given that Big phone Company charges a connection fee of $200 and $0.15 per minute. Little phone Company charges a connection fee of $175 and $0.18 per minute.

The cost for both companies for time 310 minutes will be calculated as,

Cbc= 0.15t + 200

Cbc = ( 0.15 x 310) + 200

Cbc = $246.5

The cost of little company,

Clc = 0.18t + 175

Clc = ( 0.18 x 310 ) + 175

Clp = $230.8

From the above costs, it is observed that the Little Phone Company is cheaper.

To know more about an expression follow

https://brainly.com/question/14060220

#SPJ1

A chocolate chip cookie recipe calls for 2 3/4 cups of flour. You only have a 1/4-cup measuring cup and a 3/4-cup measuring cup that you can use.

a. What are different combinations of the measuring cups that you can use to get a total of 2 3/4 cups of flour? ​

b. Write each of the combinations as an addition equation.

Answers

The different combinations of the measuring cups that you can use to get a total of 2 3/4 cups of flour include 11 cups of 1/4 or 3.67 cups of 3/4.

How to illustrate the equation?

An equation is the statement that illustrates the variables given. In this case, two or more components are taken into consideration to describe the scenario.

Since the chocolate chip cookie recipe calls for 2 3/4 cups of flour. You only have a 1/4-cup measuring cup and a 3/4-cup measuring cup that you can use.

The combination will be:.

= 2 3/4 ÷ 1/4.

= 11

The second combination will be:

= 2 3/4 ÷ 3/4

= 3.67

Learn more about equations on:

brainly.com/question/2972832

#SPJ1

In the xy-coordinate plane, the points (4, 2) and (-1, k) are on a line that is perpendicular to the line y=2x+1 . What is the value of k

Answers

A line of two points (4, 2) and (-1, k) is perpendicular to the line y=2x+1, then k = 9/2.

Given a line with equation y = mx + c, the slope is denoted by m.

Given 2 points with slopes m₁ and m₂, those two lines are perpendicular if:

m₁ x m₂ = -1

The given line equation is:  y = 2x+1

Hence,

m₁ = 2

Compute the slope from 2 points: (4, 2) and (-1, k)

m₂ = (k - 2) / (-1 - 4)

m₂ = (-1/5) (k - 2)

Use the condition for perpendicular lines:

m₁ x m₂ = -1

2 x (-1/5) (k - 2) = -1

2k - 4 = 5

2k = 9

k = 9/2

Learn more about line equation here:

https://brainly.com/question/29813143

#SPJ4

One interior angle of a triangle is 43°, and the other two angles are congruent. Choose the equation that could be used to determine the degree measure of one of the congruent angles. 2x + 43 = 180 2x − 43 = 90 x + 43 = 180 x − 43 = 90

Answers

The answer to this is 3x+56=76 hope this slays

Answer:

x − 43 = 90

Step-by-step explanation:

2x + 43 = 180

2x − 43 = 90

x + 43 = 180

x − 43 = 90

Suppose the accompanying summary statistics for a measure of social marginality for samples of youths, young adults, adults, and seniors appeared in a research paper. The social marginality score measured actual and perceived social rejection, with higher scores indicating greater social rejection. Age Group Youths Young Adults Adults Seniors
Sample Size 102 258 319 31
x :2.00 3.10 3.06 2.81
s :1.59 1.66 1.69 1.87
For purposes of this exercise, assume that it is reasonable to regard the four samples as representative of the U.S. population in the corresponding age groups and that the distributions of social marginality scores for these four groups are approximately normal with the same standard deviation. Is there evidence that the mean social marginality scores are not the same for all four age groups? Test the relevant hypotheses using α = 0.01.
Calculate the test statistic. (Round your answer to two decimal places.) F = ?
What can be said about the P-value for this test?
P-value > 0.100
0.050 < P-value < 0.100
0.010 < P-value < 0.050
0.001 < P-value < 0.010
P-value < 0.001
What can you conclude?
Reject H0. There is not convincing evidence that the mean social marginality scores are not the same for all four age groups.
Reject H0. There is convincing evidence that the mean social marginality scores are not the same for all four age groups.
Fail to reject H0. There is convincing evidence that the mean social marginality scores are not the same for all four age groups.
Fail to reject H0. There is not convincing evidence that the mean social marginality scores are not the same for all four age groups.
You may need to use the appropriate table in Appendix A to answer this question.

Answers

As per the given standard deviation, the Lana hypothesis is not equal to 25 point and the alternative hypothesis is greater.

The term standard deviation refers the measure of how dispersed the data is in relation to the mean.

Here we have given that the accompanying summary statistics for a measure of social marginality for samples of youths, young adults, adults, and seniors appeared in a research paper. The social marginality score measured actual and perceived social rejection, with higher scores indicating greater social rejection.

And we need to find if  the mean social marginality scores are not the same for all four age groups

While we looking into the given the alternate hypothesis for this problem is μ greater than 115 point.

And here we have to find the test statistics which are equal to x, power, minus μ, divided by 10 point.

And the bar is 125 points in the x bar and 25.2 points in the me.

Then it can be written as,

=> x² = 125.2

Where as 1.5 and sigma equal to 30 and then equal to 25.

Now, we have to do the Minus 115 and then divided into 30 by root 5 and 25 by root.

Then we will get it as 1.70

Now we have to simplify it.

Which is, is that greater than 1.70, which is equal to 1 minus p p, is less than 1.70, which is equal to 1 minus 0.9554, which is equal to 0.0446.

To know more about Standard deviation here

https://brainly.com/question/16555520

#SPJ4

a line through the origin, rotates around the origin in such a way that the angle, theta, between the line and the positive x-axis changes at the rate of d/dt for time > 0. Which expression gives the rate at which the slope of the line is changing

Answers

The rate at which the slope of the line is changing with respect to time is dθ/dt.

The rate at which the slope of a line is changing with respect to time is a concept in calculus that is known as the derivative. The derivative measures the rate of change of a function with respect to one of its variables. In this case, we are looking at the rate of change of the slope of the line with respect to time. The slope of a line is determined by the angle, theta, between the line and the positive x-axis. Therefore, the rate of change of the slope of the line with respect to time is equal to the rate of change of the angle, theta, with respect to time. This can be expressed as dθ/dt, where dθ represents the change in theta and dt represents the change in time. Therefore, the expression which gives the rate at which the slope of the line is changing with respect to time is dθ/dt.

Learn more about rate here

https://brainly.com/question/13481529

#SPJ4

PLEASE HELP IMMEDIATELY

Find the volume of a cylinder with a diameter of 16 inches and a height of 9 inches

What’s the volume of a NBA Basketball that has a diameter of 9.5 inches

Answers

The volume of the given cylinder is V = 1809.56 in³ and the volume of the NBA basketball is V = 448.92 in³.

What are the formulas for finding the volume of a cylinder and a sphere?

The formula for the volume of a cylinder is V = π × r² × h cubic units.

Where the radius of the cylinder is represented by 'r' and the height of the cylinder is represented by 'h'.

The formula for the volume of the sphere is V = 4/3 × π × r³ cubic units.

Where the radius of the sphere is represented by 'r'.

Calculation:

The given cylinder has a diameter is of 16 inches and a height of 9 inches.

I.e., d = 2r = 16 in and h = 9 in

Then the volume of the cylinder is

V = π × (d/2)² × h

  = π × (16/2)² × 9

  = π × 64 × 9

  = 1809.56 in³

The shape of the NBA basketball is a sphere. The given sphere has a diameter of 9.5 inches.

I.e., d = 2r = 9.5 in

Then the volume of the sphere is

V = 4/3 × π × (d/2)³

  = 4/3 × π × (9.5/2)³

  = 4/3 × π × (4.75)³

  = 4/3 × π × 107.172

  = 448.92 in³

Therefore, the volume of the cylinder and the basketball are 1809.56 in³ and 448.92 in³ respectively.

Learn more about the volume of a cylinder here:

https://brainly.com/question/24055431

#SPJ1

Suppose that the functions u and w are defined as follows.
u(x)=x² + 3
w (x)=√x+2
Find the following.
(wu) (2) =
(uw) (2) =

Answers

The values of composite functions are,

(w о u)(2) = √7 + 2

(u о w)(2) = 9 + 4√2

What is composite function?

Function composition is an operation о used in mathematics that takes two functions, f and g, and creates a function, h = g о f, such that h(x) = g. The function g is applied in this operation to the outcome of applying the function f to x.

Given:

u(x) = x^2 + 3

w(x) = √x+2

We have to find (w о u)(2) and (u о w)(2).

First to find (w о u)(x)

(w о u) = w(u(x))

           = w(x^2 + 3)

(w о u) = √(x^2 + 3) + 2

⇒ (w о u)(2) = √(2^2 + 3) + 2

   (w о u)(2) = √7 + 2

Now to find (u о w).

(u о w)(x) = u(w(x))

= u(√x+2)

= (√x+2)^2 + 3

= x + 4√x + 4 + 3

(u о w)(x) = x +  4√x + 7

⇒ (u о w)(2)  = 2 + 4√2 + 7

   (u о w)(2)  = 9 + 4√2

Hence,

The values of composite functions are,

(w о u)(2) = √7 + 2

(u о w)(2)  = 9 + 4√2

To know more about composite function, click on the link

https://brainly.com/question/10687170

#SPJ1

suppose a runner claims that her mile run time is less than 9 minutes, on average. several of her friends do not believe her, so the runner decides to do a hypothesis test, at a 5% significance level, to persuade them. she runs 7 time trials, collects the proper data, and works through the testing procedure:H0: μ=9; Ha: μ<9α=0.1 (significance level)The test statistic isz0=x¯−μ0σn√=7.7−9112√=−4.5The critical value is −z0.1=−1.28.Conclude whether to reject or not reject H0 and interpret the results.Select all that apply:Reject H0. At the 10% significance level, the test results are statistically significant to conclude that the mean time is less than 9 minutes.There is NOT sufficient evidence at the 10% significance level to conclude that the mean time is less than 9 minutes.The test statistic falls within the rejection region.The test statistic is NOT in the rejection region.

Answers

The conclusion of the hypothesis test is;

B: Reject H₀. At the 10% significance level, the data provides sufficient evidence to conclude that the mean time is less than 9 minutes.

What is the conclusion of the hypothesis test?

We are given;

Null Hypothesis; H₀: μ = 9;

Alternative Hypothesis; Hₐ: μ < 9

Significance level; α = 0.1

The critical value; z_0.1 = 1.28

Test statistic; z = -4.5

Now, the rejection rule here is that if the test statistic is below the critical value, we reject the null hypothesis at the significance level.

In this case, the the test statistic is less than the critical value and as such we reject the null hypothesis and conclude that the data provides sufficient evidence to conclude that the mean time is less than 9 minutes

Read more about Hypothesis Conclusion at; https://brainly.com/question/15980493

#SPJ1

Which is the simplified form of the expression three 7/5 X +4-232 -5/4 X

Answers

The simplified form of the expression is 123/20 x - 228

How to determine the simplified form of the expression?

From the question, we have the following parameters that can be used in our computation:

three 7/5 X +4-232 -5/4 X

Rewrite the above expression properly

So, we have the following representation

37/5 x + 4 - 232 - 5/4 x

Collect the like terms in the above expression

So, we have the following representation

37/5 x - 5/4 x + 4 - 232

Evaluate the like terms in the above expression

So, we have the following representation

123/20 x + 4 - 232

Solving further, we have

123/20 x - 228

Hence, the solution is  123/20 x - 228

Read more about simplified expression at

https://brainly.com/question/11688560

#SPJ1

Other Questions
a flooded engine occurs when a. too much fuel and not enough air reaches the engine. b. the engine stalls after you drive though standing water. c. you overfill the gas tank. d. you drive your vehicle into a canal. the regular payback method is deficient in that it does not take account of cash flows beyond the payback period. the discounted payback method corrects this fault. t or f Mr. Kenny teache a ceramic cla. She find that he ha 7/8 pound of clay for a project. The project require 1/6 of a pound. How many project can Mr. Kenny complete without running out of clay? find an equation of the tangent line to the curve at the given point. y = x3 2x + 2, (4, 58) _____is an organized group of employees who negotiate with employers about issues, such as wages and working conditions. after watching her uncle linger on life support for months jill has decided she does not want that to happen to her. what would you advise jill to do? draw all of the resonance contributors for the ortho, meta, and para benzenonium intermediates in the chlorination of benzaldehyde and explain why ortho/para substitution predominates. commercial vinegar is a 5.00% by mass aqueous solution of ethanoic acid. what is the molarity of ethanoic acid in vinegar? you may assume that the density of vinegar is 1.00 g/ml. An investigation by the Medicare Fraud Strike Force resulted in the discovery and ultimate sentencing of a registered nurse who defrauded her home health agency and the federal government. If you were the administrator of this home health agency what could you have done to prevent this individual, or any staff member, from committing fraud of this nature? What policies or procedures could you implement? What actions could you enforce, within your organization that could have uncovered this illegal behavior sooner? a popular leading indicator of economic activity is: group of answer choices stock index returns. industrial production. the prime rate of interest. commercial and industrial loan volume. Tin (II) oxide is placed into a bottle of hydrogen gas and a reaction occurs where metallic tin and water form:Skeleton Equation:Balanced Equation: Which of the following agencies provides an effective redressmechanism to financial consumers who feel have been aggrieved with conduct, products, and services?a. Securities and Exchange Commission.b. Bangko Sentral ng Pilipinasc. Bureau of Food and Drugs ayayai corporation is authorized to issue 23,500 shares of $50 par value, 10% preferred stock and 120,000 shares of $5 par value common stock. on january 1, 2022, the ledger contained the following stockholders' equity balances. $525,000 preferred stock (10,500 shares) 70,000 paid-in capital in excess of par-preferred stock 325,000 common stock (65,000 shares) 730,000 paid-in capital in excess of par-common stock retained earnings during 2022, the following transactions occurred 1 issued 2,100 shares of preferred stock for land having a fair value of $121.000, issued 1.100 shares of preferred stock for cash at $70 per share When the current in a long, straight air-filled solenoid is changing at the rate of 3000 A/S, the voltage across the solenoid is 0.600 V. The solenoid has 1200 turns and uniform cross-sectional area 35.0 mm? Assume that the magnetic field is uniform inside the solenoid and zero outside, so the inductance formula L MAN/ 2/1 for a solenoid with N turns, uniform cross-sectional area A, and length 1, applies Part A What is the magnitude B of the magnetic field in the interior of the solenoid when the current in the solenoid is 3.00 A? Express your answer with the approppriate units. ? B= Value Units Submit Request Answer Mary Ainsworth developed the Strange Situation as an assessment of parent.infant attachment. She observed that securely attached infants ____a. do not seem to care when the mother leaves the room and do not see her out when she returns b.cry if the mother leaves the room and welcome her back when she returresc. are not concerned when the mother awes, but cry to be picked up when she returns d. protest loudly when the mother leaves, but resist contact with her when she returns dr. anderson decides to search for scientific articles that are related to her topic of interest. the most efficient way for her to perform this search is to Help please help help calculate the work done by an 85.0-kg man who pushes a crate 4.00 m up along a ramp that makes an angle of with the horizontal (see below). he exerts a force of 500 n on the crate parallel to the ramp and moves at a constant speed. be certain to include the work he does on the crate and on his body to get up the ramp. one scientific theory is that at the time of the creation of the universe, that for every matter particle created, an equal amount of antimatter was created. t or f if a linear function is increasing, explain mathematically why the reciprocal function must be decreasing